4 Easy Circles, Part 2

Geometry Level 3

If the two smallest circles are congruent, and the radius of the largest circle can be expressed as a b c \dfrac{a - \sqrt b}{c} , where a , b , c a,b,c are coprime positive integers and c c is a square-free, submit a + b + c a+b+c .


The answer is 81108.

This section requires Javascript.
You are seeing this because something didn't load right. We suggest you, (a) try refreshing the page, (b) enabling javascript if it is disabled on your browser and, finally, (c) loading the non-javascript version of this page . We're sorry about the hassle.

3 solutions

David Vreken
Jun 3, 2021

Draw altitude C G CG . Since the 13 13 - 14 14 - 15 15 triangle is a 9 9 - 12 12 - 15 15 right triangle combined with a 5 5 - 12 12 - 13 13 right triangle at side 12 12 , C G = 12 CG = 12 .

Now A G C A E D \triangle AGC \sim \triangle AED and B G C B F D \triangle BGC \sim \triangle BFD by AA similarity, so let A E = 3 m AE = 3m , E D = 4 m ED = 4m , A D = 5 m AD = 5m , and r A E D = m r_{\triangle AED} = m (because it is similar to a 9 9 - 12 12 - 15 15 or 3 3 - 4 4 - 5 5 triangle, which has an inradius of r = 1 2 ( 3 + 4 5 ) = 1 r = \frac{1}{2}(3 + 4 - 5) = 1 ) and let B F = 5 n BF = 5n , D F = 12 n DF = 12n , D B = 13 n DB = 13n , and r B F D = 2 n r_{\triangle BFD} = 2n (because it is similar to a 5 5 - 12 12 - 13 13 triangle, which has an inradius of r = 1 2 ( 5 + 12 13 ) = 2 r = \frac{1}{2}(5 + 12 - 13) = 2 ).

From segment addition, A B = A D + D B = 5 m + 13 n = 14 AB = AD + DB = 5m + 13n = 14 , and since the two smallest circles are congruent, r A E D = r B F D = m = 2 n r_{\triangle AED} = r_{\triangle BFD} = m = 2n .

Solving 5 m + 13 n = 14 5m + 13n = 14 and m = 2 n m = 2n gives m = 28 23 m = \cfrac{28}{23} and n = 14 23 n = \cfrac{14}{23} .

That means D F = 12 n = 168 23 DF = 12n = \cfrac{168}{23} , F C = B C B F = 13 5 n = 229 23 FC = BC - BF = 13 - 5n = \cfrac{229}{23} , C D = D F 2 + F C 2 = ( 168 23 ) 2 + ( 229 23 ) 2 = 80554 23 CD = \sqrt{DF^2 + FC^2} = \sqrt{(\frac{168}{23})^2 + (\frac{229}{23})^2} = \cfrac{\sqrt{80554}}{23} , and the radius of the largest circle is r = 1 2 ( D F + F C C D ) = 397 + 80665 46 r = \frac{1}{2}(DF + FC - CD) = \cfrac{397 + \sqrt{80665}}{46} , so that a = 397 a = 397 , b = 80665 b = 80665 , c = 46 c = 46 , and a + b + c = 81108 a + b + c = \boxed{81108} .

A B C \triangle ABC is a compound of a 15 15 - 9 9 - 12 12 and a 13 13 - 5 5 - 12 12 right triangle sharing a common side of length 12 12 , which is the height from C of A B C \triangle ABC .
Thus, sin A = 12 15 = 4 5 \sin A=\dfrac{12}{15}=\dfrac{4}{5} \ \ \ \ \ , cos A = 9 15 = 3 5 \ \ \ \ \ \cos A=\dfrac{9}{15}=\dfrac{3}{5} \ \ \ \ \ , sin B = 12 13 \ \ \ \ \ \sin B=\dfrac{12}{13} \ \ \ \ \ and cos B = 5 13 \ \ \ \ \ \cos B=\dfrac{5}{13} .

Let A D = x AD=x . Then, D B = 14 x DB=14-x .

Moreover, E D = A D sin A E D = 4 x 5 , ED=AD\cdot \sin A\Rightarrow ED=\dfrac{4x}{5}, \ \ \ \ \ E A = A D cos A E A = 3 x 5 EA=AD\cdot \cos A\Rightarrow EA=\dfrac{3x}{5}

For the incircle of the right E A D \triangle EAD we have

r = 1 2 ( E A + E D A D ) = 1 2 ( 3 x 5 + 4 x 5 x ) = x 5 ( 1 ) r=\dfrac{1}{2}\left( EA+ED-AD \right)=\dfrac{1}{2}\left( \dfrac{3x}{5}+\dfrac{4x}{5}-x \right)=\dfrac{x}{5} \ \ \ \ \ (1) Working similarly on right F D B \triangle FDB , F D = B D sin B F D = ( 14 x ) 12 13 FD=BD\cdot \sin B\Rightarrow FD=\left( 14-x \right)\dfrac{12}{13} F B = B D cos B F B = ( 14 x ) 5 13 FB=BD\cdot \cos B\Rightarrow FB=\left( 14-x \right)\dfrac{5}{13}

r = 1 2 ( F D + F B B D ) = 1 2 ( 14 x ) ( 12 13 + 5 13 1 ) = 2 13 ( 14 x ) ( 2 ) r=\dfrac{1}{2}\left( FD+FB-BD \right)=\dfrac{1}{2}\left( 14-x \right)\left( \dfrac{12}{13}+\dfrac{5}{13}-1 \right)=\dfrac{2}{13}\left( 14-x \right) \ \ \ \ \ (2) Combining ( 1 ) (1) and ( 2 ) (2) we get x 5 = 2 13 ( 14 x ) x = 140 23 \dfrac{x}{5}=\dfrac{2}{13}\left( 14-x \right)\Leftrightarrow x=\dfrac{140}{23} Using this value, we find F D = 168 23 FD=\dfrac{168}{23} and F B = 70 23 FB=\dfrac{70}{23} Hence, by Pythagorean theorem on right C D F \triangle CDF ,

C D = F D 2 + C F 2 = ( 168 23 ) 2 + ( 13 70 23 ) 2 C D = 80665 23 CD=\sqrt{F{{D}^{2}}+C{{F}^{2}}}=\sqrt{{{\left( \dfrac{168}{23} \right)}^{2}}+{{\left( 13-\dfrac{70}{23} \right)}^{2}}}\Rightarrow CD=\dfrac{\sqrt{80665}}{23}

Finally, for the inradius R R of right C D F \triangle CDF ,

R = 1 2 ( F D + C F C D ) = 1 2 ( 168 23 + 229 23 80665 23 ) R = 397 80665 46 R=\dfrac{1}{2}\left( FD+CF-CD \right)=\dfrac{1}{2}\left( \dfrac{168}{23}+\dfrac{229}{23}-\dfrac{\sqrt{80665}}{23} \right)\Rightarrow R=\dfrac{397-\sqrt{80665}}{46} Doing analogous calculations for the inradius R {R}' of C D E \triangle CDE we find R = 373 80665 46 {R}'=\dfrac{373-\sqrt{80665}}{46} Since R < R {R}'<R , the largest circle is the incircle of C D F \triangle CDF .

For the answer, a = 397 a=397 , b = 80665 b=80665 , c = 46 c=46 , thus, a + b + c = 81108 a+b+c=\boxed{81108} .

We note that a 13-14-15 \text{13-14-15} triangle is made up of a 3-4-5 \text{3-4-5} right triangle and a 5-12-13 \text{5-12-13} right triangle sharing an altitude of 12 12 . Therefore A D E \triangle ADE is a 3-4-5 \text{3-4-5} right triangle and B D F \triangle BDF is a 5-12-13 \text{5-12-13} right triangle.

Let D B = x DB=x ; then F B = 5 13 x FB = \frac 5{13}x and D F = 12 13 x DF = \frac {12}{13}x . If r r is the radius of the two congruent incircles, then

r = 1 2 F B D F 1 2 ( F B + D F + D B ) = 2 x 13 r = \frac {\frac 12 FB \cdot DF}{\frac 12(FB+DF+DB)} = \frac {2x}{13}

Similarly, A D = 14 x AD=14-x , A E = 3 5 ( 14 x ) AE=\frac 35(14-x) . E D = 4 5 ( 14 x ) ED = \frac 45(14-x) , and

r = 1 2 A E E D 1 2 ( A E + E D + A D ) = 14 x 5 r = \frac {\frac 12 AE \cdot ED}{\frac 12(AE+ED+AD)} = \frac {14-x}5

Therefore r = 2 x 13 = 14 x 5 x = 182 23 = D B r = \dfrac {2x}{13} = \dfrac {14-x}5 \implies x = \dfrac {182}{23} = DB , D F = 12 13 D B = 168 23 DF = \dfrac {12}{13}DB = \dfrac {168}{23} , F B = 5 13 D B = 70 23 FB = \dfrac 5{13} DB = \dfrac {70}{23} , and C F = 13 F B = d f r a c 22923 CF = 13-FB = dfrac {229}{23} . The radius of the largest circle,

R = 1 2 D F C F 1 2 ( D F + C F + C D ) = D F C F D F + C F + D F 2 + C F 2 = 397 80665 46 R = \frac {\frac 12 DF \cdot CF}{\frac 12(DF+CF+CD)} = \frac {DF \cdot CF}{DF+CF+\sqrt{DF^2+CF^2}} = \frac {397-\sqrt{80665}}{46}

The required answer is a + b + c = 397 + 80665 + 46 = 81108 a+b+c = 397+80665+46 = \boxed{81108} .

0 pending reports

×

Problem Loading...

Note Loading...

Set Loading...